El "tamaño" (radio de Schwarzschild) de un agujero negro es directamente proporcional a su masa. La cifra de mérito que hay que considerar, para resolver cualquier detalle espacial, es el diámetro angular del agujero negro visto desde la Tierra. Esto se escalará como $M/d$ , donde $d$ es la distancia.
Tengo entendido que el agujero negro en el centro de M87 y Sgr A* en el centro de nuestra galaxia son los dos agujeros negros con el mayor valor de $M/d$ .
Sgr A* : $M/d \sim 4\times 10^{6}/8 = 5\times 10^{5}\ M_{\odot}$ /kpc;
M87 : $\ \ $ $M/d \sim 6\times 10^{9}/16\times 10^3 = 3.8\times 10^5\ M_{\odot}$ /kpc.
Sus alternativas sugeridas.
Andrómeda : $M/d \sim 2\times 10^8/8\times 10^2 = 2.5\times 10^5\ M_{\odot}$ /kpc;
Triángulo: ¿no tiene un agujero negro supermasivo central conocido?
Así que Andrómeda no es un objetivo loco. Su tamaño angular es sólo 2/3 del de M87. Sin embargo, otra cuestión es ¿cuántos de los 8 telescopios de la red pueden ver Andrómeda a la vez? Es imposible para el Polo Sur (como lo era M87), pero tampoco es visible durante mucho tiempo desde Chile, por lo que hay una cobertura de base reducida.
Otro argumento crucial es considerar la escala de tiempo de la variabilidad del objeto. Para poder coadjuntar imágenes, hay que asegurarse de que la imagen es estable en escalas de tiempo suficientemente largas. Pero la escala de tiempo de la variabilidad de la iluminación de acreción en un agujero negro es proporcional a su masa (véase ¿Por qué se eligió M87 para el telescopio Event Horizon en lugar de Sagitario A*? ) y esta escala de tiempo es sólo de unos 2 minutos para Sgr A* y de una hora para Andrómeda. Esto hace que los agujeros negros menos masivos sean más difíciles de visualizar con esta técnica interferométrica.
1 votos
Yo pensaría que quizás tiene que ver con que son más pequeños, pero también puede ser que no tengan discos de acreción tan activos. No lo sé, sólo estoy suponiendo.
0 votos
"Duplicado" en Astronomía : astronomy.stackexchange.com/q/30339/7209 y ver también: astronomy.stackexchange.com/q/30313/7209
0 votos
Véase también esta pregunta: physics.stackexchange.com/questions/471792/